Surface area problem in 3-d calculus

Click For Summary
The discussion revolves around finding the surface area of the equation z = (2/3)(x^(3/2) + y^(3/2)) within specified bounds. The correct approach involves using a double integral to evaluate the surface area, specifically the integral of (1+x+y)^(1/2) over the defined area. A participant initially struggled with algebraic errors, leading to incorrect answers, but ultimately confirmed the correct answer as approximately 1.4066 after resolving the mistakes. The conversation highlights the importance of careful algebraic evaluation in solving calculus problems. Overall, the thread emphasizes the process of verifying calculations to ensure accurate results in surface area problems.
dukefanfromch
Messages
2
Reaction score
0

Homework Statement



Find the area of the surface.
The surface z = (2/3)(x^(3/2) + y^(3/2)), 0 </= x </= 1, 0 </= y </= 1

Homework Equations



Double integral over S of the magnitude of dr/du cross dr/dv dS, which equals the double integral over D of the magnitude of dr/du cross dr/dv dA.
(SSs |dz/dx X dz/dy|dS = SS D |dz/dx X dz/dy|dA)

The Attempt at a Solution


_ 1 1
I found the integral to be SS D (1+x+y)^(1/2) dA = SS (1+x+y)^(1/2) dxdy
_ 0 0
but my answer keeps coming out wrong. I might be making a mistake with algebra because I get a lot of different answers when i do it different ways. This is from an NC State Calculus 3 homework assignment, if anyone may have seen this problem before and remember how to do it. Help?
 
Last edited:
Physics news on Phys.org
dukefanfromch said:

Homework Statement



Find the area of the surface.
The surface z = (2/3)(x^(3/2) + y^(3/2)), 0 </= x </= 1, 0 </= y </= 1

Homework Equations



Double integral over S of the magnitude of dr/du cross dr/dv dS, which equals the double integral over D of the magnitude of dr/du cross dr/dv dA.
(SSs |dz/dx X dz/dy|dS = SS D |dz/dx X dz/dy|dA)

The Attempt at a Solution


_ 1 1
I found the integral to be SS D (1+x+y)^(1/2) dA = SS (1+x+y)^(1/2) dxdy
_ 0 0
but my answer keeps coming out wrong. I might be making a mistake with algebra because I get a lot of different answers when i do it different ways. This is from an NC State Calculus 3 homework assignment, if anyone may have seen this problem before and remember how to do it. Help?

What you have shown is completely correct. Now, how did you do that integration and what answer did you get?
 
I got the right answer finally, 1.4066. It was an algebraic error when I was evaluating the integral. Thanks for telling me I was doing it right though, that helped me know it wasn't a mistake in my set-up.
 
Question: A clock's minute hand has length 4 and its hour hand has length 3. What is the distance between the tips at the moment when it is increasing most rapidly?(Putnam Exam Question) Answer: Making assumption that both the hands moves at constant angular velocities, the answer is ## \sqrt{7} .## But don't you think this assumption is somewhat doubtful and wrong?

Similar threads

Replies
1
Views
1K
  • · Replies 14 ·
Replies
14
Views
2K
  • · Replies 3 ·
Replies
3
Views
5K
  • · Replies 10 ·
Replies
10
Views
3K
  • · Replies 3 ·
Replies
3
Views
2K
Replies
11
Views
3K
Replies
2
Views
2K
  • · Replies 2 ·
Replies
2
Views
1K
  • · Replies 7 ·
Replies
7
Views
2K
Replies
4
Views
2K